Skip to main content
Log in

Downstream competition and upstream labor market matching

  • Original Paper
  • Published:
International Journal of Game Theory Aims and scope Submit manuscript

Abstract

This study investigates how externalities from downstream competition shape sorting in upstream labor markets. I model this as a two-stage game: A first stage of simultaneous one-to-one matching between firms and managers and a second stage of Cournot competition among matched pairs. If a firm’s technology and human capital are strategic complements, it is rational for each firm-manager pair to expect that the remaining agents will form a positive assortative matching (PAM), and the PAM on the grand market is a stable matching under rational expectations. The PAM remains stable even when they are strategic substitutes but the substitutive effect is moderate. However, if the substitutive effect is sufficiently strong, a negative assortative matching is stable. Social welfare induced by stable matchings is discussed.

This is a preview of subscription content, log in via an institution to check access.

Access this article

Price excludes VAT (USA)
Tax calculation will be finalised during checkout.

Instant access to the full article PDF.

Institutional subscriptions

Similar content being viewed by others

Notes

  1. Gudmundsson and Habis (2017) extend Sasaki and Toda (1996) work by adding transferable utilities.

  2. There is also a literature that examines externalities between couples matched to the same firm or among peers matched to the same college (Dutta and Massó 1997; Klaus and Klijn 2005; Echenique and Yenmez 2007; Pycia 2012; Kojima et al. 2013; Ashlagi et al. 2014; Inal 2015).

  3. My model is also related to that of Jehiel and Moldovanu (1996). They study an N-buyer-1-seller auction in the presence of allocation externalities. The game can be regarded as an assignment among N buyers and 1 seller.

  4. By assuming injective functions, I focus on the cases in which no agents share the same type. It reduces the exposure to equivalent matching outcomes caused by agents with the same types, but the results do not loss generality.

  5. Throughout the paper, it is implicitly assumed that \(D(y)=0\) for all \(y\ge d\). We ignore this in all proofs, because we do not loss generality.

  6. Although I focus on the cases in which the market is sufficiently large to accommodate all firms and managers, this does not cause a loss of generality. If the condition is not satisfied, there are boundary cases in which high-type firms and managers produce a large amount of the good. In this case, the price is too low for the low-type firms and managers to stay in the market. Thus, I can exclude these low-type agents from the analysis.

  7. Typically, for all \(i,j\in M\), \(\varphi _{\mu ^{M\backslash \{i,j\}}}^{\{i,j\}}(i,j)=\mu ^{M\backslash \{i,j\}}\cup \{(i,j)\}\).

  8. Sasaki and Toda consider belief correspondences, because in their setting with ordinal preferences there can be multiple stable matchings among available agents. However, in my setting with transferable utilities and heterogeneous agents, there can be at most one stable matching among available agents. Hence, I only need to consider belief functions.

  9. See Legros and Newman (2007) for example.

  10. Note that \(D\circ G^{-1}(n)-D\circ G^{-1}(0)\in (0,1)\), because \([D\circ G^{-1}]'\in (0,\frac{1}{n})\).

  11. The infimum and supremum that subject to Eq. (5).

  12. The belief function \(\varphi \) plays a key role in this problem because problems arise if there is no admissible matching where the total surplus of the agents is greater than or equal to \(z_{max}\).

  13. In contrast to assignment games with no externalities, here for a matching \(\mu ^{M_1}\cup \mu ^{M_2}\) to be \(\varphi _{\mu ^{M_1}}^{M_2}\)-stable it is necessary but not sufficient that

    1. (i)

      \(\mu ^{M_2}\) is \(\varphi _{\mu ^{M_1}}^{M_2}\)-admissible and

    2. (ii)

      \(z_{max}\le \sum _{(i,j)\in \mu ^{M_2}}\pi ^{ij}_{\mu ^{M_1}\cup \mu ^{M_2}}\).

    It is also possible that a matching that induces a less total profit is stable while one that induces a higher total profit is not.

  14. This is a standard result in assignment games with no externalities. See Legros and Newman (2007) for example.

References

  • Ashlagi I, Braverman M, Hassidim A (2014) Stability in large matching markets with complementarities. Oper Res 62(4):713–732

    Article  Google Scholar 

  • Baccara M, İmrohoroğlu A, Wilson AJ, Yariv L (2012) A field study on matching with network externalities. Am Econ Rev 102:1773–1804

    Article  Google Scholar 

  • Bando K (2012) Many-to-one matching markets with externalities among firms. J Math Econ 48(1):14–20

    Article  Google Scholar 

  • Bando K (2014) A modified deferred acceptance algorithm for many-to-one matching markets with externalities among firms. J Math Econ 52:173–181

    Article  Google Scholar 

  • Becker GS (1974) A theory of marriage. In: Economics of the family: marriage, children, and human capital. University of Chicago Press, Chicago, pp 299–351

  • Chade H, Eeckhout J (2019) Competing teams. Rev Econ Stud (forthcoming)

  • Chen B (2019) Labor market matching with ensuing competitive externalities in large economies, Available at SSRN 2868962

  • Dutta B, Massó J (1997) Stability of matchings when individuals have preferences over colleagues. J Econ Theory 75(2):464–475

    Article  Google Scholar 

  • Echenique F, Yenmez MB (2007) A solution to matching with preferences over colleagues. Games Econ Behav 59(1):46–71

    Article  Google Scholar 

  • Fisher JCD, Hafalir IE (2016) Matching with aggregate externalities. Math Soc Sci 81:1–7

    Article  Google Scholar 

  • Gudmundsson J, Habis H (2017) Assignment games with externalities revisited. Econ Theory Bull 5(2):247–257

    Article  Google Scholar 

  • Hafalir IE (2008) Stability of marriage with externalities. Int J Game Theory 37(3):353–369

    Article  Google Scholar 

  • Inal H (2015) Core of coalition formation games and fixed-point methods. Soc Choice Welf 45(4):745–763

    Article  Google Scholar 

  • Jehiel P, Moldovanu B (1996) Strategic nonparticipation. Rand J Econ 27:84–98

    Article  Google Scholar 

  • Klaus B, Klijn F (2005) Stable matchings and preferences of couples. Econ Theory 121(1):75–106

    Article  Google Scholar 

  • Kojima F, Pathak PA, Roth AE (2013) Matching with couples: stability and incentives in large markets. Q J Econ 128(4):1585–1632

    Article  Google Scholar 

  • Legros P, Newman AF (2007) Beauty is a beast, frog is a prince: assortative matching with nontransferabilities. Econometrica 75(4):1073–1102

    Article  Google Scholar 

  • Mumcu A, Saglam I (2010) Stable one-to-one matchings with externalities. Math Soc Sci 2(60):154–159

    Article  Google Scholar 

  • Pycia M (2012) Stability and preference alignment in matching and coalition formation. Econometrica 80(1):323–362

    Article  Google Scholar 

  • Pycia M, Yenmez MB (2015) Matching with externalities. Available at SSRN 2475468

  • Sasaki H, Toda M (1996) Two-sided matching problems with externalities. J Econ Theory 70(1):93–108

    Article  Google Scholar 

  • Shapley LS, Shubik M (1971) The assignment game I: the core. Int J Game Theory 1(1):111–130

    Article  Google Scholar 

  • Teytelboym A (2012) Strong stability in contractual networks and matching markets. Mimeo

  • Uetake K, Watanabe Y (2018) Entry by merger: estimates from a two-sided matching model with externalities. Available at SSRN 2188581

Download references

Author information

Authors and Affiliations

Authors

Corresponding author

Correspondence to Bo Chen.

Additional information

Publisher's Note

Springer Nature remains neutral with regard to jurisdictional claims in published maps and institutional affiliations.

I would like to thank Benny Moldovanu, Andreas Kleiner, the associate editor, and an anonymous referee for their advice. I would also like to thank Deniz Dizdar, Dominik Grafenhofer, Olga Gorelkina, Phillip Strack, Daniel Kraehmer, Huaxia Zeng, Xiaoxi Li, and seminar participants at BGSE, Spring Meeting of Young Economists, and China Meeting of the Econometric Society. Earlier versions of this paper were circulated as “Assignment Games with Externalities and Matching-based Cournot Competition” in 2013 and “Matching Prior to Cournot Competition: An Assignment Game with Externalities” in 2016. This work was supported by the National Natural Science Foundation of China under Grant No. 11701432.

Appendices

Appendix

Cournot competition

Lemma A.1

Suppose \([q\cdot D'(q)]'<0\).

  1. (i)

    The inverse function \(G^{-1}\) exists, and it is decreasing.

  2. (ii)

    \([D\circ G^{-1}]'\in (0,\frac{1}{n})\).

  3. (iii)

    For all \(x\ge 0\), \(\frac{\partial ^2 q\cdot D(q+x)}{\partial q \partial x}<0\) and \(\frac{\partial ^2 q\cdot D(q+x)}{\partial q^2}<0\).

  4. (iv)

    For all \(x\ge 0\), \(\frac{\frac{\partial ^2 q\cdot D(q+x)}{\partial q \partial x}}{\frac{\partial ^2 q\cdot D(q+x)}{\partial q^2}}\in (0,1)\).

Proof

First, because \(G'(q)=nD'(q)+[q\cdot D'(q)]'<0\), the inverse function \(G^{-1}\) exists and it is decreasing.

Second, \([D\circ G^{-1}(c)]'=\frac{D'(q)}{G'(q)}=\frac{D'(q)}{nD'(q)+[qD'(q)]'}\). The value of the last term is in between 0 and 1 / n, because \(D'(q)<0\) and \([qD'(q)]'<0\).

Third, since \([q\cdot D'(q)]'<0\) can be rewritten as \(D'(q)+q\cdot D''(q)<0\) and \(D''(q)>0\) almost everywhere, the second order derivatives of \(q\cdot D(q+x)\) are

$$\begin{aligned} \frac{\partial ^2 q\cdot D(q+x)}{\partial q \partial x}&=D'(q+x)+q\cdot D''(q+x) \nonumber \\&=[D'(q+x)+(q+x)\cdot D''(q+x)]-x\cdot D''(q+x) \nonumber \\&<0\text { for all }x\ge 0;\end{aligned}$$
(3)
$$\begin{aligned} \frac{\partial ^2 q\cdot D(q+x)}{\partial q^2}&=2D'(q+x)+q\cdot D''(q+x)\nonumber \\&=[D'(q+x)+(q+x)\cdot D''(q+x)]+D'(q+x)-x\cdot D''(q+x) \nonumber \\&<0\text { for all }x\ge 0. \end{aligned}$$
(4)

Fourth, following from Eqs. (3) and (4), \(\frac{\frac{\partial ^2 q\cdot D(q+x)}{\partial q \partial x}}{\frac{\partial ^2 q\cdot D(q+x)}{\partial q^2}}\) is larger than 0 but less than 1. \(\square \)

Proof of Lemma 1.1

In the Cournot game, given that agent pairs other than \((i,j)\in \mu \) produce in total an amount of \(Q^{-ij}\ge 0\), (ij) choose an output level \(q\ge 0\) to maximize

$$\begin{aligned} qD(q+Q^{-ij})-qc^{ij}. \end{aligned}$$

This objective function is strictly concave, because, by Lemma A.1(iii), the second order condition is satisfied:

$$\begin{aligned}&\frac{\partial ^2 qD'(q+Q^{-ij})}{\partial q^2} <0. \end{aligned}$$

The first order condition for this problem is:

$$\begin{aligned}&\frac{\partial qD(q+Q^{-ij})}{\partial q}-c^{ij}=0 \\&\quad \Leftrightarrow D(q+Q^{-ij})+ qD'(q+Q^{-ij})-c^{ij}=0.\nonumber \end{aligned}$$
(5)

Hence, in equilibrium, the output of pair (ij) is

$$\begin{aligned} q^{ij}_{\mu }&=\max \left\{ 0,~-\frac{D( Q_{\mu })-c^{ij}}{D'(Q_{\mu })}\right\} , \end{aligned}$$

where the total output under matching \(\mu \), \(Q_{\mu }\), satisfies:

$$\begin{aligned} Q_{\mu }&=-\frac{1}{D'(Q_{\mu })} \cdot \sum _{(i,j)\in \mu } \max \left\{ 0,~D(Q_{\mu })-c^{ij}\right\} \nonumber \\&\Leftrightarrow Q_{\mu }D'(Q_{\mu })+\sum _{(i,j)\in \mu } \max \left\{ 0,~D(Q_{\mu })-c^{ij}\right\} =0. \end{aligned}$$
(6)

Following from Assumption 1.1, the term on the left hand side of Eq. (6) is strictly decreasing in the value of \(Q_{\mu }\), and it is strictly positive when \(Q_{\mu }\) approached 0 and negative when \(Q_{\mu }\) approached d. Hence, following from the intermediate value theorem, there is a unique value of \(Q_{\mu }\) that solves the above equation. Equation (6) also implies

$$\begin{aligned} Q_{\mu }D'(Q_{\mu })+\sum _{(i,j)\in \mu } D(Q_{\mu })-c^{ij}\le 0 \Leftrightarrow G(Q_{\mu })\le C_{\mu } \Leftrightarrow Q_{\mu }\le G^{-1}(C_{\mu }), \end{aligned}$$
(7)

where the equalities hold if each firm produces a positive amount.

Last, I prove that in the unique equilibrium the output produced by each firm-manager pair is strictly positive. It is sufficient to show \(D(Q_{\mu })-c^{ij}>0\) for all \((i,j)\in \mu \). Because both D and \(G^{-1}\) are strictly decreasing, \(D\circ G^{-1}\) is strictly increasing and

$$\begin{aligned} D(Q_{\mu })-c^{ij}&\ge D\circ G^{-1}(C_{\mu })-c^{ij}\\&\ge D\circ G^{-1}(c^{ij})-c^{ij}\\&=[D\circ G^{-1}(1)-1]-\int _{c^{ij}}^{1}[D\circ G^{-1}(c)-c]'dc\\&>[D\circ G^{-1}(1)-1]-\int _{c^{ij}}^{1}\frac{1}{n}-1 dc\\&>0, \end{aligned}$$

where the third inequality follows from Lemma A.1(ii).

Hence, in the unique equilibrium, the total output is

$$\begin{aligned} Q_{\mu }&=G^{-1}(C_{\mu }), \end{aligned}$$

the output of pair (ij) is

$$\begin{aligned} q^{ij}_{\mu }&=-\frac{D( Q_{\mu })-c^{ij}}{D'(Q_{\mu })}, \end{aligned}$$
(8)

and the profit of (ij), obtained by substituting \(Q_{\mu }\) and \( q^{ij}_{\mu }\) back into the objective function, is

$$\begin{aligned} \pi ^{ij}_{\mu }&=-\frac{\left[ D( Q_{\mu })-c^{ij}\right] ^2}{D'(Q_{\mu })}. \end{aligned}$$

\(\square \)

Corollary A.1

For any pair (ij), \(\frac{d}{dc} \left( \frac{D\circ G^{-1}(c)-c^{ij}}{D'\circ G^{-1}(c)}\right) <0\) for all \(c\in [c^{ij},n-1+c^{ij}]\).

Proof

It is equivalent to prove that the derivative \(\frac{d}{dQ} \left( \frac{D(Q)-c^{ij}}{D'(Q)}\right) >0\) for all \(Q\in [G^{-1}(n-1+c^{ij}), G^{-1}(c^{ij})]\).

Denote \(Q^{-ij}\) as the total output of all firm-manager pairs other than (ij) and \(q^{ij}(Q^{-ij})\) as (ij)’s best response. Restrict attention to the set of \(Q^{-ij}>0\) such that \(q^{ij}(Q^{-ij})>0\). First, applying the implicit function theorem and Lemma A.1, to the first order condition (Eq. (5)), we have

$$\begin{aligned} \frac{dq^{ij}}{d Q^{-ij}}&=-\frac{\frac{\partial ^2 q^{ij}D(q^{ij}+Q^{-ij})}{\partial q^{ij}\partial Q^{-ij}}}{\frac{\partial ^2 q^{ij}D(q^{ij}+Q^{-ij})}{\partial (q^{ij})^2}}\in (-1,0). \end{aligned}$$

Therefore, \(q^{ij}(Q^{-ij})+Q^{-ij}\) is strictly increasing in \(Q^{-ij}\) for \(Q^{-ij}\ge 0\), with the infimum achieved when \(Q^{-ij}=0\) and the supremum achieved when \(q^{ij}=0\).Footnote 11 In the Cournot equilibrium under any matching, every firm-manager pair produces a positive amount. Hence, the values of \(G^{-1}(n-1+c^{ij})\) and \(G^{-1}(c^{ij})\) lie between the infimum and supremum.

Second, taking the derivative with respect to \(Q^{-ij}\) on both sides of Eq. (8) yields

$$\begin{aligned} \frac{dq^{ij}}{dQ^{-ij}} =&-\frac{d\left[ \frac{ D(q^{ij}+Q^{-ij})-c^{ij}}{ D'(q^{ij}+Q^{-ij}) }\right] }{d(q^{ij}+Q^{-ij})}\cdot \frac{d (q^{ij}+Q^{-ij})}{Q^{-ij}}\\ =&-\frac{d\left[ \frac{ D(q^{ij}+Q^{-ij})-c^{ij}}{ D'(q^{ij}+Q^{-ij}) }\right] }{d(q^{ij}+Q^{-ij})}\cdot \left( \frac{d q^{ij}}{dQ^{-ij}}+1\right) <0. \end{aligned}$$

Hence, \(\frac{d\left[ \frac{ D(q^{ij}+Q^{-ij})-c^{ij}}{ D'(q^{ij}+Q^{-ij}) }\right] }{d(q^{ij}+Q^{-ij})}>0 \) for all \(q^{ij}(Q^{-ij})+Q^{-ij}\in [G^{-1}(n-1+c^{ij}),G^{-1}(c^{ij})] \). This yields the desired result. \(\square \)

Stable matchings

1.1 Linear programming

Proof of Lemma 1.2

The proof uses Shapley and Shubik (1971) linear programming method:

$$\begin{aligned} \begin{aligned}&{\text {(Primal)}}\\&\max _x z= \sum _{i\in I_s} \sum _{j\in J_s} \pi ^{ij}_{\varphi _{\mu ^{M_1}}^{M_2}(i,j)}\cdot x_{ij}\\&s.t.~\textstyle \sum _{i\in I_2} x_{ij}\le 1 ,~~\forall j\in J_2\\&\textstyle \sum _{j\in J_2} x_{ij}\le 1 ,~~\forall i\in I_2\\&x_{ij}\ge 0 \end{aligned} \quad \begin{aligned}&{\text {(Dual)}}\\&\min _{u,v} w=\sum _{i\in I_2} u_i +\sum _{j\in J_2} v_j\\&s.t.~u_i+v_j\ge \pi ^{ij}_{\varphi _{\mu ^{M_1}}^{M_2}(i,j)},~~\forall i\in I_2 ,\forall j\in J_2\\&u_i\ge 0 \\&v_j\ge 0 \end{aligned} \end{aligned}$$

The maximum value \(z_{max}\) of the primal problem is attained when \(x_{ij}=0\) or 1 for all i and j. By strong duality, both the primal and the dual have the same optimal value, i.e., \(z_{max}=w_{min}\). By complementary slackness, if \(x^*_{ij}=1\) then the corresponding dual constraint must be tight which implies \(u_i+v_j= \pi ^{ij}_{\varphi _{\mu ^{M_1}}^{M_2}(i,j)}\).

Taking an extreme solution \(x^*\) to the primal problem, I construct a matching \(\mu ^{M_2}\) such that it satisfies that

$$\begin{aligned} (i,j)\in \mu ^{M_2}\quad \textit{ if and only if }\quad x^*_{ij}=1. \end{aligned}$$

(Sufficiency). Take a solution \((u^*,v^*)^{M_2}\) to the dual problem. If \(\mu ^{M_2}\) is admissible, then the triplet \((\mu ^{M_2},(u^*,v^*)^{M_2})\) satisfies the conditions in Definition 1.1, and thus \(\mu ^{M_1}\cup \mu ^{M_2}\) is a \(\varphi _{\mu ^{M_1}}^{M_2}\)-stable matching.Footnote 12\(^,\)Footnote 13

(Necessity). Take a matching \(\mu ^{M_2}\). If condition ii of Lemma 1.2 is not satisfied, the matching is not stable. If condition i is not satisfied, then \(V(\mu ^{M_2}| \varphi _{\mu ^{M_1}}^{M_2})<z_{max}=w_{min}\). Therefore, the feasibility condition of Definition 1.1 cannot be satisfied, and thus \(\mu ^{M_2}\) is not stable. \(\square \)

1.2 Stable matchings under rational beliefs

Due to externalities from the downstream, it is quite involving to prove Theorem 2.1. In particular, in case i PAM results in the highest industry production efficiency and thus the lowest Cournot equilibrium market price, which is a counterforce of stability (because it requires the highest total expected profit). This conflict makes the proof difficult. Similarly, this difficulty also exists in case (iii). Case ii has no such conflict and thus is easier to prove.

To prove cases (i) and (iii), I provide a series of technical lemmas below. The general idea is to prove these two cases in a recursive method, a series of pairwise permutations (Lemma B.7). I want to show that the total expected profit under a permuted matching is greater than that under the original one. However, the profit of a firm-manager pair is proportional to the square of their unit profit, and I need to make comparisons across the square terms. In order to do so, I factorize differences in the square terms (Lemmas B.5B.6) and make comparisons across the unit profits (Lemmas B.2B.4).

Since an expected profit crucially depends on its associated expected industry production efficiency, the first step is to make comparisons across expected industry production efficiencies (Lemma B.1).

Now, consider a partition \([M_1,M_2]\) with \(\mu ^{M_1}\) on market \(M_1\). Denote

$$\begin{aligned} M_2=&I_2\cup J_2:=\{i_1,\ldots ,i_{t}\}\cup \{j_1,\ldots ,j_{t}\}. \end{aligned}$$

Denote \(\mu _P^{M_2\backslash \{i,j \}}\) as the positively ordered matching (based on indicators) on market \(M_2\backslash \{i,j \}\) such that for all \((i_x,j_z),(i_{x'},j_{z'})\in \mu _P^{M_2\backslash \{i,j \}}, x>x'\Leftrightarrow z>z'\). In short, let us denote \(C^{ij}:=C_{\mu ^{M_1}\cup (i,j)\cup \mu _P^{M_2\backslash \{i,j \}}}\). Focus on the two scenarios below for Lemmas B.1B.7:

  • Scenario 1: \(\frac{\partial ^2 c}{\partial f \partial s}\le 0\); \(f_{i_k}>f_{i_{k+1}}~\text {and}~s_{i_k}>s_{i_{k+1}}\) for \(k=1,\ldots ,t-1\).

  • Scenario 2: \(\frac{\partial ^2 c}{\partial f \partial s}> 0\); \(f_{i_k}>f_{i_{k+1}}~\text {and}~s_{i_k}<s_{i_{k+1}}\) for \(k=1,\ldots ,t-1\).

Lemma B.1

Take three numbers, kpq, such that \(1\le k<p, q\le t\). In Scenarios 1 and 2, the four industry production efficiency differences, \(C^{i_kj_k}-C^{i_kj_q} \), \(C^{i_pj_q}-C^{i_pj_k}\), \(C^{i_pj_q}-C^{i_pj_k}\), and \(C^{i_pj_q}-C^{i_kj_q}\), are all non-positive.

Proof

Let us first consider the case in which \(p\le q\). The industrial production efficiencies under four matchings are:

$$\begin{aligned} C^{i_kj_k}&=C_{\mu ^{M_1}\cup (i_k,j_k)\cup \mu _P^{M_2\backslash \{i_k,j_k \}}} =\sum _{l=1}^{t}c^{i_lj_l} +C_{\mu ^{M_1}};\\ C^{i_kj_q}&=C_{\mu ^{M_1}\cup (i_k,j_q)\cup \mu _P^{M_2\backslash \{i_k,j_q \}}} =\sum _{l=1}^{k-1}c^{i_lj_l} +c^{i_kj_q} +\sum _{l=k+1}^{q}c^{i_{l}j_{l-1}} \\&\quad +\sum _{l=q+1}^{t}c^{i_lj_l} +C_{\mu ^{M_1}}; \\ C^{i_pj_q}&=C_{\mu ^{M_1}\cup (i_p,j_q)\cup \mu _P^{M_2\backslash \{i_p,j_q \}}} =\sum _{l=1}^{p-1}c^{i_lj_l} +c^{i_pj_q} +\sum _{l=p+1}^{q}c^{i_{l}j_{l-1}} \\&\quad +\sum _{l=q+1}^{t}c^{i_lj_l} +C_{\mu ^{M_1}} ; \\ C^{i_pj_k}&=C_{\mu ^{M_1}\cup (i_p,j_k)\cup \mu _P^{M_2\backslash \{i_p,j_k \}}} = \sum _{l=1}^{k-1}c^{i_lj_l} +c^{i_pj_k}\\&\quad +\sum _{l=k+1}^{p}c^{i_{l-1}j_{l}} +\sum _{l=p+1}^{t}c^{i_lj_l} +C_{\mu ^{M_1}}. \end{aligned}$$

In both scenarios:

$$\begin{aligned} C^{i_kj_k}-C^{i_kj_q}&=\sum _{l=1}^{t}c^{i_lj_l} -\left[ \sum _{l=1}^{k-1}c^{i_lj_l} +c^{i_kj_q} +\sum _{l=k+1}^{q}c^{i_{l}j_{l-1}} +\sum _{l=q+1}^{t}c^{i_lj_l} \right]&\nonumber \\&=(c^{i_kj_k}-c^{i_kj_q}) +\sum _{l=k+1}^{q}(c^{i_{l}j_{l}}- c^{i_{l}j_{l-1}})&\end{aligned}$$
(9)
$$\begin{aligned}&=\int ^{s_{j_k}}_{s_{j_q}}\frac{\partial c(f_{i_k},s)}{\partial s}ds -\sum _{l=k+1}^{q}\int ^{s_{j_{l-1}}}_{s_{j_l}}\frac{\partial c(f_{i_{l}},s)}{\partial s}ds \nonumber \\&=\sum _{l=k+1}^{q}\int ^{s_{j_{l-1}}}_{s_{j_l}}\left[ \frac{\partial c(f_{i_k},s)}{\partial s}-\frac{\partial c(f_{i_{l}},s)}{\partial s}\right] ds&\nonumber \\&=\sum _{l=k+1}^{q}\int ^{s_{j_{l-1}}}_{s_{j_l}}\int _{f_{i_l}}^{f_{i_k}}\frac{\partial ^2 c(f,s)}{\partial f \partial s}dfds&\nonumber \\&\le 0; \end{aligned}$$
(10)
$$\begin{aligned} C^{i_pj_q} - C^{i_pj_k}&= (c^{i_pj_q}-c^{i_pj_k}) + \sum _{l=k+1}^{p}(c^{i_{l-1}j_{l-1}}-c^{i_{l-1}j_{l}} ) + \sum _{l=p+1}^{q}(c^{i_{l}j_{l-1}}-c^{i_{l}j_{l}}) \end{aligned}$$
(11)
$$\begin{aligned}&=-\int _{s_{j_q}}^{s_{j_k}}\frac{\partial c(f_{i_p},s)}{\partial s}ds + \sum _{l=k+1}^{p}\int _{s_{j_l}}^{s_{j_{l-1}}}\frac{\partial c(f_{i_{l-1}},s)}{\partial s}ds \nonumber \\&\quad + \sum _{l=p+1}^{q}\int _{s_{j_l}}^{s_{j_{l-1}}}\frac{\partial c(f_{i_l},s)}{\partial s}ds \nonumber \\&= \sum _{l=k+1}^{p}\int _{s_{j_l}}^{s_{j_{l-1}}}\frac{\partial c(f_{i_{l-1}},s)}{\partial s}-\frac{\partial c(f_{i_p},s)}{\partial s}ds \nonumber \\&\quad + \sum _{l=p+1}^{q}\int _{s_{j_l}}^{s_{j_{l-1}}}\frac{\partial c(f_{i_l},s)}{\partial s}-\frac{\partial c(f_{i_p},s)}{\partial s}ds \nonumber \\&= \sum _{l=k+1}^{p}\int _{s_{j_l}}^{s_{j_{l-1}}}\int _{f_{i_p}}^{f_{i_{l-1}}}\frac{\partial ^2 c(f,s)}{\partial f \partial s}dfds\nonumber \\&\quad + \sum _{l=p+1}^{q}\int _{s_{j_l}}^{s_{j_{l-1}}}\int _{f_{i_p}}^{f_{i_{l}}}\frac{\partial ^2 c(f,s)}{\partial f \partial s}dfds\nonumber \\&\le 0; \end{aligned}$$
(12)
$$\begin{aligned} C^{i_kj_k}-C^{i_pj_k}&=\sum _{l=k}^pc^{i_lj_l}-\left[ c^{i_pj_k}+ \sum _{l=k+1}^pc^{i_{l-1}j_l} \right] \nonumber \\&=(c^{i_kj_k}-c^{i_pj_k})+\frac{1}{n}\sum _{l=k+1}^p(c^{i_lj_l}-c^{i_{l-1}j_l}) \end{aligned}$$
(13)
$$\begin{aligned}&=\int _{f_{i_p}}^{f_{i_k}}c(f,s_{j_k})df -\sum _{l=k+1}^p\int _{f_{i_l}}^{f_{i_{l-1}}}c(f,s_{j_l})df \nonumber \\&=\sum _{l=k+1}^p\int _{f_{i_l}}^{f_{i_{l-1}}}\int _{s_{j_l}}^{s_{j_k}}\frac{\partial ^2 c(f,s)}{\partial f \partial s}dsdf\nonumber \\&\le 0; \end{aligned}$$
(14)
$$\begin{aligned} C^{i_pj_q}-C^{i_kj_q}&=\left[ \sum _{l=k}^{p-1}c^{i_lj_l}+c^{i_pj_q}\right] -\left[ c^{i_kj_q}+\sum _{l=k+1}^{p}c^{i_lj_{l-1}}\right] \\&=-(c^{i_kj_q}-c^{i_pj_q})+\sum _{l=k+1}^{p}( c^{i_{l-1}j_{l-1}}-c^{i_lj_{l-1}}) \\&=-\int ^{f_{i_k}}_{f_{i_p}}\frac{\partial c(f,s_{j_q})}{\partial f}df+\sum _{l=k+1}^{p}\int ^{f_{i_{l-1}}}_{f_{i_l}}\frac{\partial c(f,s_{j_{l-1}})}{\partial f}df \\&=\sum _{l=k+1}^{p}\int ^{f_{i_{l-1}}}_{f_{i_l}}\left[ \frac{\partial c(f,s_{j_{l-1}})}{\partial f}-\frac{\partial c(f,s_{j_q})}{\partial f}\right] df \\&=\sum _{l=k+1}^{p}\int ^{f_{i_{l-1}}}_{f_{i_l}}\int _{s_{j_q}}^{s_{j_{l-1}}}\frac{\partial ^2 c(f,s)}{\partial f \partial s}dsdf. \\&\le 0. \end{aligned}$$

The proof for the case in which \(p\ge q\) is just a mirror image and thus is omitted. \(\square \)

Denote the unit profit of (ij) in a Nash equilibrium of the Cournot game, given a matching \(\mu ^{M_1}\cup (i,j)\cup \mu _P^{M_2\backslash \{i,j \}}\), by

$$\begin{aligned} y^{ij}:=D\circ G^{-1}(C^{ij})-c^{ij}. \end{aligned}$$

Lemma B.2

In Scenario 1, the three unit profit differences, \(y^{i_kj_k}-y^{i_kj_q}\), \(y^{i_kj_k}-y^{i_pj_k}\), \(y^{i_kj_q}-y^{i_pj_q}\), and \(y^{i_pj_k}-y^{i_pj_q}\), are all strictly positive.

Proof

This proof is an application of Lemmas A.1(ii) and B.1. Let us first consider the case in which \(p\le q\). First,

$$\begin{aligned} y^{i_kj_k}-y^{i_kj_q}&=D\circ G^{-1}(C^{i_kj_k}) -D\circ G^{-1}(C^{i_kj_q}) -(c^{i_kj_k}-c^{i_kj_q})\\&\ge \frac{1}{n}(C^{i_kj_k}-C^{i_kj_q} ) -(c^{i_kj_k}-c^{i_kj_q})\\&= -\frac{1}{n}\sum _{l=k+1}^{q}\int ^{s_{j_{l-1}}}_{s_{j_l}}\frac{\partial c(i_{l},s)}{\partial s}ds-\frac{n-1}{n}\int ^{s_{j_k}}_{s_{j_q}}\frac{\partial c(f_{i_k},s)}{\partial s}ds\\&>0, \end{aligned}$$

where the first inequality follows from Lemma A.1(ii) and \(C_P^{i_kj_k}-C_P^{i_kj_q}\le 0\) and the second equation follows from Eqs. (9) and (10). Similarly,

$$\begin{aligned} y^{i_kj_k}-y^{i_pj_k}&=D\circ G^{-1}(C^{i_kj_k}) -D\circ G^{-1}(C^{i_pj_k}) -(c^{i_kj_k}-c^{i_pj_k})\\&\ge \frac{1}{n}(C^{i_kj_k}- C^{i_pj_k}) -(c^{i_kj_k}-c^{i_pj_k})\\&=-\frac{1}{n}\sum _{l=k+1}^p\int _{f_{i_l}}^{f_{i_{l-1}}}\frac{\partial c(f,s_{j_l})}{\partial f}df -\frac{n-1}{n}\int _{f_{i_p}}^{f_{i_k}}\frac{\partial c(f,s_{j_k})}{\partial f}df\\&>0, \end{aligned}$$

where the first inequality follows from Lemma A.1(ii) and \(C_P^{i_kj_k}-C_P^{i_pj_k}\le 0\) and the second equation following from Eqs. (13) and (14). Third,

$$\begin{aligned} y^{i_kj_q}-y^{i_pj_q}&=D\circ G^{-1}(C^{i_kj_q})- D\circ G^{-1}(C^{i_pj_q}) -(c^{i_kj_q}-c^{i_pj_q})\\&\ge -(c^{i_kj_q}-c^{i_pj_q})\\&=-\int _{f_{i_p}}^{f_{i_k}}\frac{\partial c(f,s_{j_q})}{\partial f}df\\&>0, \end{aligned}$$

where the first inequality follows from Lemma A.1(ii) and \(C_P^{i_pj_q}-C_P^{i_kj_q}\le 0\). Fourth,

$$\begin{aligned} y^{i_pj_k}-y^{i_pj_q}&=D\circ G^{-1}(C^{i_pj_k})- D\circ G^{-1}(C^{i_pj_q}) -(c^{i_pj_k}-c^{i_pj_q})\\&\ge -(c^{i_pj_k}-c^{i_pj_q})\\&=-\int _{s_{j_q}}^{s_{j_q}}\frac{\partial c(f_{i_p},s)}{\partial s}ds\\&>0, \end{aligned}$$

where the first inequality follows from Lemma A.1(ii) and \(C_P^{i_pj_q}-C_P^{i_pj_k}\le 0\).

The proof for the case in which \(p\ge q\) is just a mirror image and thus is omitted. \(\square \)

Lemma B.3

In Scenarios 1 and 2, for \(n\ge 2\),

$$\begin{aligned} (y^{i_kj_k}-y^{i_kj_q})+(y^{i_pj_q}-y^{i_pj_k}) \ge -\frac{n-2}{n}\int ^{f_{i_k}}_{f_{i_p}}\int ^{s_{j_k}}_{s_{j_q}}\frac{\partial ^2 c(f,s)}{\partial f \partial s}dsdf \ge 0. \end{aligned}$$

Proof

$$\begin{aligned}&\left( y^{i_kj_k}-y^{i_kj_q}\right) +\left( y^{i_pj_q}-y^{i_pj_k}\right) \\&\quad = \left\{ \left[ D\circ G^{-1}(C^{i_kj_k})-c^{i_kj_k}\right] - \left[ D\circ G^{-1}(C^{i_kj_q})-c^{i_kj_q}\right] \right\} \\&\qquad +\left\{ \left[ D\circ G^{-1}(C^{i_pj_q})-c^{i_pj_q}\right] - \left[ D\circ G^{-1}(C^{i_pj_k})-c^{i_pj_k}\right] \right\} \\&\quad \ge \left[ \frac{1}{n}(C^{i_kj_k}-C^{i_kj_q} ) -(c^{i_kj_k}-c^{i_kj_q})\right] + \left[ \frac{1}{n}(C^{i_pj_q} - C^{i_pj_k}) - ( c^{i_pj_q}-c^{i_pj_k})\right] \\&\quad = -\frac{1}{n}\sum _{l=k+1}^{q}\int ^{s_{j_{l-1}}}_{s_{j_l}}\frac{\partial c(f_{i_{l}},s)}{\partial s}ds\\&\qquad +\frac{1}{n}\sum _{l=k+1}^{p}\int _{s_{j_l}}^{s_{j_{l-1}}}\frac{\partial c(f_{i_{l-1}},s)}{\partial s}ds +\frac{1}{n}\sum _{l=p+1}^{q}\int _{s_{j_l}}^{s_{j_{l-1}}}\frac{\partial c(f_{i_l},s)}{\partial s}ds\\&\qquad -\frac{n-1}{n}\int ^{s_{j_k}}_{s_{j_q}}\left[ \frac{\partial c(f_{i_k},s)}{\partial s} - \frac{\partial c(f_{i_p},s)}{\partial s} \right] ds \\&\quad =\frac{1}{n}\sum _{l=k+1}^{p}\int _{f_{i_{l}}}^{f_{i_{l-1}}}\int _{s_{j_l}}^{s_{j_{l-1}}}\frac{\partial ^2 c(f,s)}{\partial f \partial s}dsdf -\frac{n-1}{n}\int ^{f_{i_k}}_{f_{i_p}}\int ^{s_{j_k}}_{s_{j_q}}\frac{\partial ^2 c(f,s)}{\partial f \partial s}dsdf\\&\quad \ge -\frac{n-2}{n}\int ^{f_{i_k}}_{f_{i_p}}\int ^{s_{j_k}}_{s_{j_q}}\frac{\partial ^2 c(f,s)}{\partial f \partial s}dsdf \\&\quad \ge 0 \text { for }n\ge 2, \end{aligned}$$

where the first inequality follows from Lemma A.1(ii), \(C_P^{i_kj_k}-C_P^{i_kj_q}\le 0\), and \(C^{i_pj_q}-C^{i_pj_k}\le 0\). \(\square \)

Lemma B.4

In Scenario 2, \(y^{i_pj_q}-y^{i_pj_k}>0\).

Proof

For the case in which \(p\le q\):

$$\begin{aligned} y^{i_pj_q}-y^{i_pj_k}&= D\circ G^{-1}\left( C^{i_pj_q}\right) - D\circ G^{-1}(C^{i_pj_k}) -\left( c^{i_pj_q}-c^{i_pj_k}\right) \\&\ge \frac{1}{n}\left[ C^{i_pj_q}-C^{i_pj_k}\right] -(c^{i_pj_q}-c^{i_pj_k})\\&=\frac{1}{n}\sum _{l=k+1}^{p}(c^{i_{l-1}j_{l-1}}-c^{i_{l-1}j_{l}} ) + \frac{1}{n}\sum _{l=p+1}^{q}(c^{i_{l}j_{l-1}}-c^{i_{l}j_{l}})\\&\quad +\frac{n-1}{n}\left( c^{i_pj_k} - c^{i_pj_q} \right) \\&= \frac{1}{n}\sum _{l=k+1}^{p}\int _{s_{j_l}}^{s_{j_{l-1}}}\frac{\partial c(f_{i_{l-1}},s)}{\partial s}ds + \frac{1}{n}\sum _{l=p+1}^{q}\int _{s_{j_l}}^{s_{j_{l-1}}}\frac{\partial c(f_{i_l},s)}{\partial s}ds\\&\quad +\frac{n-1}{n}\int _{s_{j_q}}^{s_{j_k}}\frac{\partial c(f_{i_p},s)}{\partial s}ds\\&>0, \end{aligned}$$

where the first inequality follows from Lemma A.1(ii) and \(C^{i_pj_q}-C^{i_pj_k}\le 0\) and the second equality follows from Eqs. (11) and (12). The proof for the case in which \(p\ge q\) is the same as above except for that there is no summation from \(p+1\) to q. \(\square \)

Lemma B.5

In Scenario 1,

$$\begin{aligned} (y^{i_kj_k})^2+(y^{i_pj_q})^2-(y^{i_kj_q})^2-(y^{i_pj_k})^2 > 0. \end{aligned}$$

Proof

$$\begin{aligned}&(y^{i_kj_k})^2+(y^{i_pj_q})^2-(y^{i_kj_q})^2-(y^{i_pj_k})^2\\&\quad =(y^{i_kj_k}-y^{i_kj_q})(y^{i_kj_k}+y^{i_kj_q})+(y^{i_pj_q}-y^{i_pj_k})(y^{i_pj_q}+y^{i_pj_k}) \\&\quad > \left[ (y^{i_kj_k}-y^{i_kj_q})+(y^{i_pj_q}-y^{i_pj_k})\right] (y^{i_pj_q}+y^{i_pj_k})\\&\quad \ge 0, \end{aligned}$$

where the first inequality follows from Lemma B.2 and the second inequality follows from Lemma B.3. \(\square \)

Lemma B.6

In Scenario 2, if \(\frac{\partial ^2 c^2}{\partial f \partial s}/\frac{\partial ^2 c}{\partial f \partial s}\le \frac{n-2}{n}\cdot 2D\circ G^{-1}(0)\), then

$$\begin{aligned} (y^{i_kj_k})^2+(y^{i_pj_q})^2-(y^{i_kj_q})^2-(y^{i_pj_k})^2 \ge 0. \end{aligned}$$

Proof

First,

$$\begin{aligned}&(y^{i_kj_k})^2-(y^{i_kj_q})^2\\&\quad =\left( \left[ D\circ G^{-1}(C^{i_kj_k})-c^{i_kj_k}\right] -\left[ D\circ G^{-1}(C^{i_kj_q})-c^{i_kj_q}\right] \right) \\&\qquad \cdot \left( \left[ D\circ G^{-1}(C^{i_kj_k})-c^{i_kj_k}\right] +\left[ D\circ G^{-1}(C^{i_kj_q})-c^{i_kj_q}\right] \right) \\&\quad \ge ( y^{i_kj_k}-y^{i_kj_q})\cdot \left[ D\circ G^{-1}(C^{i_kj_k})+D\circ G^{-1}(C^{i_kj_q})\right] +\left[ (c^{i_kj_k})^2-(c^{i_kj_q})^2\right] , \end{aligned}$$

where the inequality follows from \(C^{i_kj_k}-C^{i_kj_q}\le 0\). Second,

$$\begin{aligned}&(y^{i_pj_q})^2-(y^{i_pj_k})^2\\&\quad = \left( \left[ D\circ G^{-1}(C^{i_pj_q})-c^{i_pj_q}\right] -[D\circ G^{-1}(C^{i_pj_k})-c^{i_pj_k}]\right) \\&\qquad \cdot \left( \left[ D\circ G^{-1}(C^{i_pj_q})-c^{i_pj_q}\right] + \left[ D\circ G^{-1}(C^{i_pj_k})-c^{i_pj_k}\right] \right) \\&\quad \ge (y^{i_pj_q}-y^{i_pj_k})\cdot \left[ D\circ G^{-1}(C^{i_pj_q})+D\circ G^{-1}(C^{i_pj_k})\right] +\left[ (c^{i_pj_q})^2-(c^{i_pj_k})^2\right] \\&\quad \ge (y^{i_pj_q}-y^{i_pj_k})\cdot \left[ D\circ G^{-1}(C^{i_kj_k})+D\circ G^{-1}(C^{i_kj_q})\right] +\left[ (c^{i_pj_q})^2-(c^{i_pj_k})^2\right] , \end{aligned}$$

where the first inequality follows from \(C^{i_pj_q}-C^{i_pj_k}\le 0\) and \(c^{i_pj_q}<c^{i_pj_k}\) and the second inequality follows from equations \(C^{i_kj_q}-C_P^{i_pj_q}\le 0\), \(C^{i_kj_k}-C_P^{i_pj_k}\le 0\), and \(y^{i_pj_q}-y^{i_pj_k}>0\) (Lemma B.4).

Sum up the two components:

$$\begin{aligned}&(y^{i_kj_k})^2+(y^{i_pj_q})^2-(y^{i_kj_q})^2-(y^{i_pj_k})^2\\&\quad \ge \left[ (y^{i_kj_k}-y^{i_kj_q})+(y^{i_pj_q}-y^{i_pj_k})\right] \cdot \left[ D\circ G^{-1}(C^{i_kj_k})+D\circ G^{-1}(C^{i_kj_q})\right] \\&\qquad +(c^{i_kj_k})^2-(c^{i_k\mu _k(i_k)})^2+(c^{i_pj_q})^2-(c^{i_pj_k})^2\\&\quad \ge -\frac{n-2}{n}\int ^{f_{i_k}}_{f_{i_p}}\int ^{s_{j_k}}_{s_{j_q}}\frac{\partial ^2 c(f,s)}{\partial f \partial s}dsdf \cdot 2D\circ G^{-1}(0)\\&\qquad +\int ^{f_{i_k}}_{f_{i_p}}\int ^{s_{j_k}}_{s_{j_q}}\frac{\partial ^2[c(f,s)]^2}{\partial f \partial s}(f,s)dsdf\\&\quad \ge 0~\text { if }\frac{\partial ^2 c^2}{\partial f \partial s}/\frac{\partial ^2 c}{\partial f \partial s}\le \frac{n-2}{n}\cdot 2D\circ G^{-1}(0), \end{aligned}$$

where the second inequality follows from Lemma B.3. \(\square \)

Lemma B.7

In Scenario 1,

$$\begin{aligned} \sum _{(i,j)\in \mu ^{M_2}_{P}} \left[ D\circ G^{-1}(C^{ij})-c^{ij}\right] ^2 - \sum _{(i,j)\in \mu ^{M_2}} \left[ D\circ G^{-1}(C^{ij})-c^{ij}\right] ^2> 0. \end{aligned}$$

In Scenario 2, if \(\frac{\partial ^2 c^2}{\partial f \partial s}/\frac{\partial ^2 c}{\partial f \partial s}\le \frac{n-2}{n}\cdot 2D\circ G^{-1}(0)\),

$$\begin{aligned} \sum _{(i,j)\in \mu ^{M_2}_{P}} \left[ D\circ G^{-1}(C^{ij})-c^{ij}\right] ^2 - \sum _{(i,j)\in \mu ^{M_2}} \left[ D\circ G^{-1}(C^{ij})-c^{ij}\right] ^2\ge 0. \end{aligned}$$

Proof

First, define a sequence of permutated matchings of \(\mu ^{M_2}\), \(\{\mu _1,\mu _2,\ldots ,\mu _t\}\):

$$\begin{aligned} \mu _1&:=\mu ^{M_2};\\ \mu _{k+1}&:=\left\{ (i_k,j_k),(\mu _k(j_k),\mu _k(i_k)) \right\} \cup \mu _k\backslash \left\{ (i_k,\mu _k(i_k)),(\mu _k(j_k),j_k)\right\} \\&\quad \quad \text { for }\quad k=1,2,\ldots ,t-1;\\ \mu _t&:=\{(i_1,j_1),(i_2,j_2),\ldots ,(i_t,j_t) \}. \end{aligned}$$

Then,

$$\begin{aligned}&\sum _{(i,j)\in \mu ^{M_2}_{P}} \left[ D\circ G^{-1}(C^{ij})-c^{ij}\right] ^2 - \sum _{(i,j)\in \mu ^{M_2}} [D\circ G^{-1}(C^{ij})-c^{ij}]^2\\&\quad = \sum _{k=1}^{t-1}\left[ \sum _{(i,j)\in \mu ^{M_2}_{k+1}}[D\circ G^{-1}(C^{ij})-c^{ij}]^2 - \sum _{(i,j)\in \mu ^{M_2}_{k}}[D\circ G^{-1}(C^{ij})-c^{ij}]^2 \right] \\&\quad = \sum _{k=1}^{t-1}\left[ \left[ D\circ G^{-1}(C^{i_kj_k})-c^{i_kj_k}\right] ^2+\left[ D\circ G^{-1}(C^{\mu _k(j_k)\mu _k(i_k)})-c^{\mu _k(j_k)\mu _k(i_k)}\right] ^2\right. \\&\qquad \left. - \left[ D\circ G^{-1}(C^{i_k\mu _k(i_k)})-c^{i_k\mu _k(i_k)}\right] ^2 -\left[ D\circ G^{-1}(C^{\mu _k(j_k)j_k})-c^{\mu _k(j_k)j_k}\right] ^2 \right] \\&\quad = \sum _{k=1}^{t-1} \left( y^{i_kj_k}\right) ^2+\left( y^{\mu _{k}(j_k)\mu _k(i_k)}\right) ^2-\left( y^{i_k\mu _k(i_k)}\right) ^2-\left( y^{\mu _{k}(j_k)j_k}\right) ^2\\&\quad \ge 0~(>0 \text { in Scenario 1}) , \end{aligned}$$

where the last inequality follows from Lemmas B.5 and B.6 . \(\square \)

Proof of Theorem 2.1

To prove the theorem, it is sufficient to check that the conditions in Lemma 1.2 are satisfied. I only consider assortative matchings among available agents, and thus \(\varphi \)-admissibility is automatically satisfied. I only need to check condition i. Again, consider a partition \([M_1,M_2]\) with \(\mu ^{M_1}\) on market \(M_1\), where

$$\begin{aligned} M_2=&I_2\cup J_2:=\{i_1,\ldots ,i_{t}\}\cup \{j_1,\ldots ,j_{t}\}. \end{aligned}$$

Take a matching \(\mu ^{M_2}\) on market \(M_2\). For cases i and ii, the conditional beliefs that I propose are \(\varphi _{\mu ^{M_1}}^{M_2}(i,j)=\mu ^{M_1}\cup \{ (i,j) \}\cup \mu _+^{M_1\backslash \{i,j\}}\), and it is sufficient to show that \(V(\mu ^{M_2}_+|\varphi _{\mu ^{M_1}}^{M_2})>V(\mu ^{M_2}|\varphi _{\mu ^{M_1}}^{M_2})\) if \(\mu ^{M_2}\ne \mu ^{M_2}_+\). For case iii, the conditional beliefs that I propose are \(\varphi _{\mu ^{M_1}}^{M_2}(i,j)=\mu ^{M_1}\cup \{ (i,j) \}\cup \mu _-^{M_1\backslash \{i,j\}}\), and it is sufficient to show that \(V(\mu ^{M_2}_-|\varphi _{\mu ^{M_1}}^{M_2})>V(\mu ^{M_2}|\varphi _{\mu ^{M_1}}^{M_2})\) if \(\mu ^{M_2}\ne \mu ^{M_2}_-\).

In the following, I will prove cases i and iii first, because these two cases are similar, and prove case ii afterward.

Case (i) Take a matching \(\mu ^{M_2}\ne \mu ^{M_2}_+\). Without loss of generality, let us focus on Scenario 1 as above.

$$\begin{aligned}&V\left( \mu ^{M_2}_+|\varphi _{\mu ^{M_1}}^{M_2}\right) -V\left( \mu ^{M_2}|\varphi _{\mu ^{M_1}}^{M_2}\right) \\&\quad =\sum _{(i,j)\in \mu ^{M_2}_{+}}\pi ^{ij}_{\mu ^{M_1}\cup \{(i,j)\}\cup \mu _+^{M_2\backslash \{i,j\}}}- \sum _{(i,j)\in \mu ^{M_2}}\pi ^{ij}_{\mu ^{M_1}\cup \{(i,j)\}\cup \mu _+^{M_2\backslash \{i,j\}}}\\&\quad = \sum _{(i,j)\in \mu ^{M_2}_{+}} \frac{[D\circ G^{-1}(C^{ij})-c^{ij}]^2}{-D'\circ G^{-1}(C^{ij})} - \sum _{(i,j)\in \mu ^{M_2}} \frac{[D\circ G^{-1}(C^{ij})-c^{ij}]^2}{-D'\circ G^{-1}(C^{ij})}\\&\quad \ge \sum _{(i,j)\in \mu ^{M_2}_{+}} \frac{[D\circ G^{-1}(C^{ij})-c^{ij}]^2}{-D'\circ G^{-1}\left( C_{\mu ^{M_1}\cup \mu _+^{M_2}}\right) } - \sum _{(i,j)\in \mu ^{M_2}} \frac{[D\circ G^{-1}(C^{ij})-c^{ij}]^2}{-D'\circ G^{-1}\left( C_{\mu ^{M_1}\cup \mu _+^{M_2}}\right) }\\&\quad >0. \end{aligned}$$

The first inequality follows from \(C_{\mu ^{M_1}\cup \mu ^{M_2}_+}- C_{\mu ^{M_1}\cup \mu ^{M_2}}\le 0\), because \(\frac{\partial ^2 c}{\partial f \partial s}\le 0\).Footnote 14 The second inequality follows from Lemma B.7. Hence, the rational beliefs contain PAMs among available agents and \(\mu _+\) is stable under rational beliefs.

Case (iii) Take a matching \(\mu ^{M_2}\ne \mu ^{M_2}_-\). Without loss of generality, let us focus on Scenario 2 as above.

$$\begin{aligned}&V(\mu ^{M_2}_-|\varphi _{\mu ^{M_1}}^{M_2})-V(\mu ^{M_2}|\varphi _{\mu ^{M_1}}^{M_2})\\&\quad =\sum _{(i,j)\in \mu ^{M_2}_{-}}\pi ^{ij}_{\mu ^{M_1}\cup \{(i,j)\}\cup \mu _-^{M_2\backslash \{i,j\}}}- \sum _{(i,j)\in \mu ^{M_2}}\pi ^{ij}_{\mu ^{M_1}\cup \{(i,j)\}\cup \mu _-^{M_2\backslash \{i,j\}}}\\&\quad = \sum _{(i,j)\in \mu ^{M_2}_{-}} \frac{[D\circ G^{-1}(C^{ij})-c^{ij}]^2}{-D'\circ G^{-1}(C^{ij})} - \sum _{(i,j)\in \mu ^{M_2}} \frac{[D\circ G^{-1}(C^{ij})-c^{ij}]^2}{-D'\circ G^{-1}(C^{ij})}\\&\quad > \sum _{(i,j)\in \mu ^{M_2}_{-}} \frac{[D\circ G^{-1}(C^{ij})-c^{ij}]^2}{-D'\circ G^{-1}(C^{ij})} - \sum _{(i,j)\in \mu ^{M_2}} \frac{[D\circ G^{-1}(C^{ij})-c^{ij}]^2}{-D'\circ G^{-1}(C^{ij})}\\&\quad \ge 0. \end{aligned}$$

The first inequality follows from \(C_{\mu ^{M_1}\cup \mu ^{M_2}_-}- C_{\mu ^{M_1}\cup \mu ^{M_2}}< 0\), because \(\frac{\partial ^2 c}{\partial f \partial s}> 0\). The second inequality follows from Lemma B.7. Hence, the rational beliefs contain NAMs among available agents \(\mu _-\) is stable under rational beliefs.

Case (ii) Again, suppose \(\mu ^{M_2}\ne \mu ^{M_2}_+\), and without loss of generality, let us focus on Scenario 1 as above. First,

$$\begin{aligned} V\left( \mu ^{M_2}_+|\varphi _{\mu ^{M_1}}^{M_2}\right)&=\sum _{(i,j)\in \mu ^{M_2}_+}\pi ^{ij}_{\mu ^{M_1}\cup \mu _+^{M_2}}\\&=\sum _{(i,j)\in \mu ^{M_2}_+}-\frac{\left[ D\circ G^{-1}(C_{\mu ^{M_1}\cup \mu _+^{M_2}})-c^{ij}\right] ^2}{D'\circ G^{-1}(C_{\mu ^{M_1}\cup \mu _+^{M_2}})};\\ V\left( \mu ^{M_2}|\varphi _{\mu ^{M_1}}^{M_2}\right)&=\sum _{(i,j)\in \mu ^{M_2}}\pi ^{ij}_{\mu ^{M_1}\cup \{(i,j)\}\cup \mu _+^{M_2\backslash \{i,j\}}}\\&=\sum _{(i,j)\in \mu ^{M_2}}-\frac{\left[ D\circ G^{-1}(C_{\mu ^{M_1}\cup \{(i,j)\}\cup \mu _+^{M_2\backslash \{i,j\}}})-c^{ij}\right] ^2}{D'\circ G^{-1}(C_{\mu ^{M_1}\cup \{(i,j)\}\cup \mu _+^{M_2\backslash \{i,j\}}})}\\&<\sum _{(i,j)\in \mu ^{M_2}}-\frac{D\circ G^{-1}(C_{\mu ^{M_1}\cup \{(i,j)\}\cup \mu _+^{M_2\backslash \{i,j\}}})-c^{ij}}{D'\circ G^{-1}(C_{\mu ^{M_1}\cup \{(i,j)\}\cup \mu _+^{M_2\backslash \{i,j\}}})}\\&\quad \cdot \left[ D\circ G^{-1}(C_{\mu ^{M_1}\cup \mu _+^{M_2}})-c^{ij}\right] \\&<\sum _{(i,j)\in \mu ^{M_2}}-\frac{\left[ D\circ G^{-1}(C_{\mu ^{M_1}\cup \mu _+^{M_2}})-c^{ij}\right] ^2}{D'\circ G^{-1}(C_{\mu ^{M_1}\cup \mu _+^{M_2}})}. \end{aligned}$$

The first inequality follows from \(C_{\mu ^{M_1}\cup \mu _+^{M_2}}>C_{\mu ^{M_1}\cup \mu ^{M_2}}\) for all \(\mu ^{M_2}\ne \mu ^{M_2}_+\), because \(\frac{\partial ^2 c}{\partial f \partial s}>0\). The second inequality follows from Corollary A.1. The condition of the corollary is satisfied because for all \(1\le p,q\le t\),

$$\begin{aligned} (n-1)+c^{i_pj_q}\ge (n-1)+\min \left\{ c^{i_pj_p},c^{i_qj_q}\right\} \ge C_{\mu ^{M_1}}+\sum _{k=1}^tc^{i_kj_k}=C_{\mu ^{M_1}\cup \mu _+^{M_2}}. \end{aligned}$$

Second, the prescribed condition of case (ii) implies

$$\begin{aligned}&\frac{\partial ^2}{\partial f \partial s}\left[ D\circ G^{-1}(C_{\mu ^{M_1}\cup \mu _+^{M_2}})-c(f,s)\right] ^2>0. \end{aligned}$$

As a result in standard assignment games with no externality, this supermodularity implies

$$\begin{aligned} \sum _{(i,j)\in \mu ^{M_2}_+}\left[ D\circ G^{-1}(C_{\mu ^{M_1}\cup \mu _+^{M_2}})-c^{ij}\right] ^2>\sum _{(i,j)\in \mu ^{M_2}}\left[ D\circ G^{-1}(C_{\mu ^{M_1}\cup \mu _+^{M_2}})-c^{ij}\right] ^2 . \end{aligned}$$

Hence, \(V(\mu ^{M_2}_+|\varphi _{\mu ^{M_1}}^{M_2})>V(\mu ^{M_2}|\varphi _{\mu ^{M_1}}^{M_2})\). Thus, again, the rational beliefs contain PAMs among available agents and \(\mu _+\) is stable under rational beliefs. \(\square \)

Social welfare

Proof of Proposition 2.2

For a given matching \(\mu \), the consumer surplus is

$$\begin{aligned} CS(\mu ):=\int _{0}^{Q_{\mu }}D(q)dq -D(Q_{\mu })Q_{\mu } \end{aligned}$$

and the producer surplus is

$$\begin{aligned} PS(\mu ):=D(Q_{\mu })Q_{\mu }-\sum _{(i,j)\in \mu }c^{ij}q^{ij}_{\mu }. \end{aligned}$$

Then, the total surplus is then \(TS(\mu ):=CS(\mu )+PS(\mu )\).

Suppose \((i_1,j_1),(i_2,j_2)\in \mu \). Take another matching \(\mu '\) such that \((i_1,j_2),(i_2,j_1) \in \mu '\) and \(\mu '\backslash \{(i_1,j_2),(i_2,j_1) \}\)=\(\mu \backslash \{(i_1,j_1),(i_2,j_2) \}\). With out loss of generality, assume \(i_1>i_2\) and \(j_1>j_2\). Here, I only need to prove that \(TS(\mu )>TS(\mu ')\) for cases 1 and 2 and that \(TS(\mu )<TS(\mu ')\) for case 3.

Case 1 Let \(M_1\), \(M_2\), and \(\mu ^{M_1}\) be \(M\backslash \{i_1,i_2,j_1,j_2\}\), \(\{i_1,i_2,j_1,j_2\}\), and \(\mu \cap \mu '\), respectively. On the one hand, following from the proof of case (i) of Theorem 2.1, we have

$$\begin{aligned} PS(\mu )=&\sum _{(i,j)\in \mu ^{M_1}}\pi ^{ij}_{\mu ^{M_1}\cup \mu _+^{M_2}} +\sum _{(i,j)\in \mu ^{M_2}_{+}}\pi ^{ij}_{\mu ^{M_1}\cup \mu _+^{M_2}}\\ >&\sum _{(i,j)\in \mu ^{M_1}}\pi ^{ij}_{\mu ^{M_1}\cup \mu _-^{M_2}} +\sum _{(i,j)\in \mu ^{M_2}_-}\pi ^{ij}_{\mu ^{M_1}\cup \mu _-^{M_2}}=PS(\mu '). \end{aligned}$$

On the other hand, because \(C_{\mu }<C_{\mu '}\) and, thus, \(Q_{\mu }>Q_{\mu '}\), we have

$$\begin{aligned} CS(\mu )=&\int _{0}^{Q_{\mu }}D(q)dq -D(Q_{\mu })Q_{\mu }\\ =&\int _{0}^{Q_{\mu }}D(q)-D(Q_{\mu })dq \\>&\int _{0}^{Q_{\mu }}D(q)-D(Q_{\mu '})dq \\ >&\int _{0}^{Q_{\mu '}}D(q)-D(Q_{\mu '})dq =CS(\mu '). \end{aligned}$$

Hence, in this case \(TS(\mu )>TS(\mu ')\).

Case 2 Here, \(C_{\mu }>C_{\mu '}\) and thus \(Q_{\mu }<Q_{\mu '}\). The difference between the total surpluses across the two matchings is

$$\begin{aligned}&TS(\mu )-TS(\mu ')\\&\quad =\int _{Q_{\mu '}}^{Q_{\mu }}D(q)dq -\left[ \sum _{(i,j)\in \mu }c^{ij}q^{ij} -\sum _{(i,j)\in \mu '}c^{ij}q^{ij} \right] \\&\quad>D(Q_{\mu '})[Q_{\mu }-Q_{\mu '}] -\left[ \sum _{(i,j)\in \mu }c^{ij}q^{ij} -\sum _{(i,j)\in \mu '}c^{ij}q^{ij} \right] \\&\quad =\sum _{(i,j)\in \mu }[D(Q_{\mu '})-c^{ij}]\cdot \frac{D\circ G^{-1}(C_{\mu })-c^{ij}}{-D'\circ G^{-1}(C_{\mu })}\\&\qquad -\sum _{(i,j)\in \mu '}[D(Q_{\mu '})-c^{ij}]\cdot \frac{D\circ G^{-1}(C_{\mu '})-c^{ij}}{-D'\circ G^{-1}(C_{\mu '})}\\&\quad >\sum _{(i,j)\in \mu }[D(Q_{\mu '})-c^{ij}]\cdot \frac{D\circ G^{-1}(C_{\mu '})-c^{ij}}{-D'\circ G^{-1}(C_{\mu '})}\\&\qquad -\sum _{(i,j)\in \mu '}\left[ D(Q_{\mu '})-c^{ij}\right] \cdot \frac{D\circ G^{-1}(C_{\mu '})-c^{ij}}{-D'\circ G^{-1}(C_{\mu '})}\\&\quad =\frac{\left[ D\circ G^{-1}(C_{\mu '})-c^{i_1j_1}\right] ^2+\left[ D\circ G^{-1}(C_{\mu '})-c^{i_2j_2}\right] ^2 }{-D'\circ G^{-1}(C_{\mu '})}\\&\qquad -\frac{\left[ D\circ G^{-1}(C_{\mu '})-c^{i_1j_2}\right] ^2+\left[ D\circ G^{-1}(C_{\mu '})-c^{i_2j_1}\right] ^2 }{-D'\circ G^{-1}(C_{\mu '})}\\&\quad =\frac{-2D'\circ G^{-1}(C_{\mu '})\left[ c^{ij}+c^{i'j'}-c^{i'j}-c^{ij'}\right] +\left[ (c^{ij})^2+(c^{i'j'})^2-(c^{i'j})^2-(c^{ij'})^2\right] }{-D'\circ G^{-1}(C_{\mu '})}\\&\quad =\frac{1}{-D'\circ G^{-1}(C_{\mu '})}\int _{f_{i'}}^{f_i}\int _{s_{j'}}^{s_j}-2D'\circ G^{-1}(C_{\mu '})\frac{\partial ^2 c(f,s)}{\partial f \partial s}+\frac{\partial ^2[c(f,s)]^2}{\partial f\partial s}dsdf\\&\quad \ge 0, \end{aligned}$$

where the second inequality follows from Corollary A.1. Hence, for any matching \(\mu '\ne \mu _+\), there is a matching \(\mu \) such that \(TS(\mu )>TS(\mu ')\). Therefore, \(\mu _+\) is efficient.

Case 3 This proof is the same as that of case 1 except that the inequalities hold in the opposite directions. In particular, \(PS(\mu )<PS(\mu ')\) follows from the rationality requirement. Thus, \(TS(\mu )<TS(\mu ')\). Therefore, the negative assortative matching is efficient. \(\square \)

Rights and permissions

Reprints and permissions

About this article

Check for updates. Verify currency and authenticity via CrossMark

Cite this article

Chen, B. Downstream competition and upstream labor market matching. Int J Game Theory 48, 1055–1085 (2019). https://doi.org/10.1007/s00182-019-00690-4

Download citation

  • Accepted:

  • Published:

  • Issue Date:

  • DOI: https://doi.org/10.1007/s00182-019-00690-4

Keywords

JEL classification

Navigation